HW3, Chapter 3, Problem 4, MA453, Fall 2008, Prof. Walther

Problem Statement

Could somebody please state the problem?


Discussion

How do you prove that an element and its inverse have the same order? I understand the idea but do not know how to prove it.

-Wooi-Chen


I thought this worked as a proof.

$ g^k=1 $ element g having order of k

$ (g^k)^{-1}=(1)^{-1} $

$ g^{-k}=1 $

$ (g^{-1})^k=1 $ inverse of g having order of k

This could be wrong, but it makes sense.

-Daniel


That actually makes sense to me as well. It is kind of playing with the order which power comes, that's the idea I get.

-Wooi-Chen


I think if you prove its cyclic the inverse will always be the same

-Matt


in this case, there are two cases where g has finite/infinite order of k.

- Panida


Daniel, have we defined what it is to take an inverse function? I am not sure if that is ok, to just take the inverse of both sides. And, can we bring the inverse in from the power? I am pretty sure it is ok to have the inverse of g^k is equal to the inverse of one, however I am not sure it is trivial to get that the inverse of g to the k is equal to one from this.

-Allen


Back to HW3

Back to MA453 Fall 2008 Prof. Walther

Alumni Liaison

Have a piece of advice for Purdue students? Share it through Rhea!

Alumni Liaison